When is $p^2+1$ twice of a prime?












6












$begingroup$


When trying to solve a bigger problem, I came across the problem of characterising all primes $p,q$ such that $p^2+1=2q$. That is, are there necessary and sufficient conditions for primes $p,q$ to satisfy the equation? Even better, is there a paramatrisation of solutions (though, this is probably unlikely)? I know the solutions $(p,q)=(3,5),(5,13),(11,61),(19,181),(29,421)$ but I don't know that these are the only ones (indeed, it seems likely there are many more).



Some incomplete thoughts: I immediately made the factorisation $(p+1)(p-1)=2(q-1)$, and because $p$ is obviously odd, let $p=2p_1+1$. This leads us to the equation $2p_1(p_1+1)=q-1$. If we substituted $q=2q_1+1$ we have $p_1(p_1+1)=q_1$, so $q$ is twice of the product of two consecutive integers, plus one. It follows that $q$ is $1$ mod $4$, and $p^2$ is $1$ mod $8$. The same conclusion can be obtained simply by noting that $-1$ must be a quadratic residue mod $2q$, hence $1$ is a $4$th power residue mod $2q$. By Lagrange's Theorem we have $4midphi(2q)=q-1$, so $q equiv1$ mod $4$. Any further thoughts are appreciated!










share|cite|improve this question











$endgroup$








  • 7




    $begingroup$
    oeis.org/A048161
    $endgroup$
    – mathlove
    Dec 29 '18 at 5:51






  • 3




    $begingroup$
    You should definitely look at the link in the first comment. Whether there are infinitely many such $p$ is unknown... BTW if $p>5$ then $pequiv pm 1 mod 10,$ for if $5<pequiv pm 3 mod 10$ then $ ( p^2+1)/2$ is greater than $5$ and divisible by $5.$
    $endgroup$
    – DanielWainfleet
    Dec 29 '18 at 6:28












  • $begingroup$
    @mathlove Wow, thanks. If you make that an answer I would happily accept it :)
    $endgroup$
    – YiFan
    Dec 29 '18 at 8:50










  • $begingroup$
    @YiFan Do you want to find all the solutions or only decide whether infinite many exist ? Besides some basic restrictions, there won't be useful methods. You will just have to check the number $frac{p^2+1}{2}$ in general. If you are interested in an evidence that infinite many solutions probably exist, I can undelete my answer.
    $endgroup$
    – Peter
    Dec 30 '18 at 10:44












  • $begingroup$
    @Peter I wanted to see if there was a nice necessary and sufficient criterion that could be used to describe all solutions to the equation, but I now know that's probably not going to happen. I'm certainly interested in anything related to the problem though, and your answer is welcome :)
    $endgroup$
    – YiFan
    Dec 30 '18 at 11:21
















6












$begingroup$


When trying to solve a bigger problem, I came across the problem of characterising all primes $p,q$ such that $p^2+1=2q$. That is, are there necessary and sufficient conditions for primes $p,q$ to satisfy the equation? Even better, is there a paramatrisation of solutions (though, this is probably unlikely)? I know the solutions $(p,q)=(3,5),(5,13),(11,61),(19,181),(29,421)$ but I don't know that these are the only ones (indeed, it seems likely there are many more).



Some incomplete thoughts: I immediately made the factorisation $(p+1)(p-1)=2(q-1)$, and because $p$ is obviously odd, let $p=2p_1+1$. This leads us to the equation $2p_1(p_1+1)=q-1$. If we substituted $q=2q_1+1$ we have $p_1(p_1+1)=q_1$, so $q$ is twice of the product of two consecutive integers, plus one. It follows that $q$ is $1$ mod $4$, and $p^2$ is $1$ mod $8$. The same conclusion can be obtained simply by noting that $-1$ must be a quadratic residue mod $2q$, hence $1$ is a $4$th power residue mod $2q$. By Lagrange's Theorem we have $4midphi(2q)=q-1$, so $q equiv1$ mod $4$. Any further thoughts are appreciated!










share|cite|improve this question











$endgroup$








  • 7




    $begingroup$
    oeis.org/A048161
    $endgroup$
    – mathlove
    Dec 29 '18 at 5:51






  • 3




    $begingroup$
    You should definitely look at the link in the first comment. Whether there are infinitely many such $p$ is unknown... BTW if $p>5$ then $pequiv pm 1 mod 10,$ for if $5<pequiv pm 3 mod 10$ then $ ( p^2+1)/2$ is greater than $5$ and divisible by $5.$
    $endgroup$
    – DanielWainfleet
    Dec 29 '18 at 6:28












  • $begingroup$
    @mathlove Wow, thanks. If you make that an answer I would happily accept it :)
    $endgroup$
    – YiFan
    Dec 29 '18 at 8:50










  • $begingroup$
    @YiFan Do you want to find all the solutions or only decide whether infinite many exist ? Besides some basic restrictions, there won't be useful methods. You will just have to check the number $frac{p^2+1}{2}$ in general. If you are interested in an evidence that infinite many solutions probably exist, I can undelete my answer.
    $endgroup$
    – Peter
    Dec 30 '18 at 10:44












  • $begingroup$
    @Peter I wanted to see if there was a nice necessary and sufficient criterion that could be used to describe all solutions to the equation, but I now know that's probably not going to happen. I'm certainly interested in anything related to the problem though, and your answer is welcome :)
    $endgroup$
    – YiFan
    Dec 30 '18 at 11:21














6












6








6





$begingroup$


When trying to solve a bigger problem, I came across the problem of characterising all primes $p,q$ such that $p^2+1=2q$. That is, are there necessary and sufficient conditions for primes $p,q$ to satisfy the equation? Even better, is there a paramatrisation of solutions (though, this is probably unlikely)? I know the solutions $(p,q)=(3,5),(5,13),(11,61),(19,181),(29,421)$ but I don't know that these are the only ones (indeed, it seems likely there are many more).



Some incomplete thoughts: I immediately made the factorisation $(p+1)(p-1)=2(q-1)$, and because $p$ is obviously odd, let $p=2p_1+1$. This leads us to the equation $2p_1(p_1+1)=q-1$. If we substituted $q=2q_1+1$ we have $p_1(p_1+1)=q_1$, so $q$ is twice of the product of two consecutive integers, plus one. It follows that $q$ is $1$ mod $4$, and $p^2$ is $1$ mod $8$. The same conclusion can be obtained simply by noting that $-1$ must be a quadratic residue mod $2q$, hence $1$ is a $4$th power residue mod $2q$. By Lagrange's Theorem we have $4midphi(2q)=q-1$, so $q equiv1$ mod $4$. Any further thoughts are appreciated!










share|cite|improve this question











$endgroup$




When trying to solve a bigger problem, I came across the problem of characterising all primes $p,q$ such that $p^2+1=2q$. That is, are there necessary and sufficient conditions for primes $p,q$ to satisfy the equation? Even better, is there a paramatrisation of solutions (though, this is probably unlikely)? I know the solutions $(p,q)=(3,5),(5,13),(11,61),(19,181),(29,421)$ but I don't know that these are the only ones (indeed, it seems likely there are many more).



Some incomplete thoughts: I immediately made the factorisation $(p+1)(p-1)=2(q-1)$, and because $p$ is obviously odd, let $p=2p_1+1$. This leads us to the equation $2p_1(p_1+1)=q-1$. If we substituted $q=2q_1+1$ we have $p_1(p_1+1)=q_1$, so $q$ is twice of the product of two consecutive integers, plus one. It follows that $q$ is $1$ mod $4$, and $p^2$ is $1$ mod $8$. The same conclusion can be obtained simply by noting that $-1$ must be a quadratic residue mod $2q$, hence $1$ is a $4$th power residue mod $2q$. By Lagrange's Theorem we have $4midphi(2q)=q-1$, so $q equiv1$ mod $4$. Any further thoughts are appreciated!







abstract-algebra number-theory prime-numbers diophantine-equations






share|cite|improve this question















share|cite|improve this question













share|cite|improve this question




share|cite|improve this question








edited Jan 7 at 10:13







YiFan

















asked Dec 29 '18 at 4:54









YiFanYiFan

4,7531727




4,7531727








  • 7




    $begingroup$
    oeis.org/A048161
    $endgroup$
    – mathlove
    Dec 29 '18 at 5:51






  • 3




    $begingroup$
    You should definitely look at the link in the first comment. Whether there are infinitely many such $p$ is unknown... BTW if $p>5$ then $pequiv pm 1 mod 10,$ for if $5<pequiv pm 3 mod 10$ then $ ( p^2+1)/2$ is greater than $5$ and divisible by $5.$
    $endgroup$
    – DanielWainfleet
    Dec 29 '18 at 6:28












  • $begingroup$
    @mathlove Wow, thanks. If you make that an answer I would happily accept it :)
    $endgroup$
    – YiFan
    Dec 29 '18 at 8:50










  • $begingroup$
    @YiFan Do you want to find all the solutions or only decide whether infinite many exist ? Besides some basic restrictions, there won't be useful methods. You will just have to check the number $frac{p^2+1}{2}$ in general. If you are interested in an evidence that infinite many solutions probably exist, I can undelete my answer.
    $endgroup$
    – Peter
    Dec 30 '18 at 10:44












  • $begingroup$
    @Peter I wanted to see if there was a nice necessary and sufficient criterion that could be used to describe all solutions to the equation, but I now know that's probably not going to happen. I'm certainly interested in anything related to the problem though, and your answer is welcome :)
    $endgroup$
    – YiFan
    Dec 30 '18 at 11:21














  • 7




    $begingroup$
    oeis.org/A048161
    $endgroup$
    – mathlove
    Dec 29 '18 at 5:51






  • 3




    $begingroup$
    You should definitely look at the link in the first comment. Whether there are infinitely many such $p$ is unknown... BTW if $p>5$ then $pequiv pm 1 mod 10,$ for if $5<pequiv pm 3 mod 10$ then $ ( p^2+1)/2$ is greater than $5$ and divisible by $5.$
    $endgroup$
    – DanielWainfleet
    Dec 29 '18 at 6:28












  • $begingroup$
    @mathlove Wow, thanks. If you make that an answer I would happily accept it :)
    $endgroup$
    – YiFan
    Dec 29 '18 at 8:50










  • $begingroup$
    @YiFan Do you want to find all the solutions or only decide whether infinite many exist ? Besides some basic restrictions, there won't be useful methods. You will just have to check the number $frac{p^2+1}{2}$ in general. If you are interested in an evidence that infinite many solutions probably exist, I can undelete my answer.
    $endgroup$
    – Peter
    Dec 30 '18 at 10:44












  • $begingroup$
    @Peter I wanted to see if there was a nice necessary and sufficient criterion that could be used to describe all solutions to the equation, but I now know that's probably not going to happen. I'm certainly interested in anything related to the problem though, and your answer is welcome :)
    $endgroup$
    – YiFan
    Dec 30 '18 at 11:21








7




7




$begingroup$
oeis.org/A048161
$endgroup$
– mathlove
Dec 29 '18 at 5:51




$begingroup$
oeis.org/A048161
$endgroup$
– mathlove
Dec 29 '18 at 5:51




3




3




$begingroup$
You should definitely look at the link in the first comment. Whether there are infinitely many such $p$ is unknown... BTW if $p>5$ then $pequiv pm 1 mod 10,$ for if $5<pequiv pm 3 mod 10$ then $ ( p^2+1)/2$ is greater than $5$ and divisible by $5.$
$endgroup$
– DanielWainfleet
Dec 29 '18 at 6:28






$begingroup$
You should definitely look at the link in the first comment. Whether there are infinitely many such $p$ is unknown... BTW if $p>5$ then $pequiv pm 1 mod 10,$ for if $5<pequiv pm 3 mod 10$ then $ ( p^2+1)/2$ is greater than $5$ and divisible by $5.$
$endgroup$
– DanielWainfleet
Dec 29 '18 at 6:28














$begingroup$
@mathlove Wow, thanks. If you make that an answer I would happily accept it :)
$endgroup$
– YiFan
Dec 29 '18 at 8:50




$begingroup$
@mathlove Wow, thanks. If you make that an answer I would happily accept it :)
$endgroup$
– YiFan
Dec 29 '18 at 8:50












$begingroup$
@YiFan Do you want to find all the solutions or only decide whether infinite many exist ? Besides some basic restrictions, there won't be useful methods. You will just have to check the number $frac{p^2+1}{2}$ in general. If you are interested in an evidence that infinite many solutions probably exist, I can undelete my answer.
$endgroup$
– Peter
Dec 30 '18 at 10:44






$begingroup$
@YiFan Do you want to find all the solutions or only decide whether infinite many exist ? Besides some basic restrictions, there won't be useful methods. You will just have to check the number $frac{p^2+1}{2}$ in general. If you are interested in an evidence that infinite many solutions probably exist, I can undelete my answer.
$endgroup$
– Peter
Dec 30 '18 at 10:44














$begingroup$
@Peter I wanted to see if there was a nice necessary and sufficient criterion that could be used to describe all solutions to the equation, but I now know that's probably not going to happen. I'm certainly interested in anything related to the problem though, and your answer is welcome :)
$endgroup$
– YiFan
Dec 30 '18 at 11:21




$begingroup$
@Peter I wanted to see if there was a nice necessary and sufficient criterion that could be used to describe all solutions to the equation, but I now know that's probably not going to happen. I'm certainly interested in anything related to the problem though, and your answer is welcome :)
$endgroup$
– YiFan
Dec 30 '18 at 11:21










2 Answers
2






active

oldest

votes


















4












$begingroup$

We have infinitely many primes of the form $$frac{p^2+1}{2}$$ where $p$ is itself prime, if we have infinite many positive integers $k$ such that $$2k+1$$ and $$2k^2+2k+1$$ are simultaneously prime. (in this case, just set $p=2k+1$). The Bunyakovsky conjecture implies that this is the case, so very likely infinitely many examples exist. But I am convinced that the problem is open.






share|cite|improve this answer











$endgroup$





















    1












    $begingroup$

    I propose the following "kind of parametrization". Remark first, as you did, that necessarily $qequiv 1$ mod $4$ (this can be seen directly from the characterization of integers which are sums of squares, see e.g. Samuel's ANT, V-6). Then work in the Gauss ring $mathbf Z[i]$, which is a PID with units $u=pm 1,pm i$. For convenience, write $z'≈z$ for $z'=uz$. It is classically known that $2$ is totally ramified and $q$ splits completely in the Gauss ring, more precisely $2=(1+i)^2≈(1+i)(1-i)$ and $q≈(x+iy)(x-iy)$, where $xpm iy$ are prime elements of $mathbf Z[i]$. Because of unique factorization (up to units), the given equation will be equivalent to $p+i≈(1+i)(x+iy)$, hence our "pseudo-parametrization" will be as follows: 1) Take a prime $qequiv 1$ mod $4$ and decompose it in $mathbf Z[i]$; 2) Solve $p+i≈(1+i)(x+iy)$ in $mathbf Z^2$. Because of symmetry, let us solve only the equation $p+i=-(1+i)(x+iy)$. By identification, $p=y-x$ and $-1=x+y$, so $p=2y+1$. It remains however to check that $2y+1$ is a prime in $mathbf Z$, which is why our method cannot be considered as a genuine parametrization. Although we have just shown that the OP question is essentially equivalent to the Bunyakovsky conjecture evoked by @Peter, I don't know if this gives any new hint.






    share|cite|improve this answer











    $endgroup$













      Your Answer





      StackExchange.ifUsing("editor", function () {
      return StackExchange.using("mathjaxEditing", function () {
      StackExchange.MarkdownEditor.creationCallbacks.add(function (editor, postfix) {
      StackExchange.mathjaxEditing.prepareWmdForMathJax(editor, postfix, [["$", "$"], ["\\(","\\)"]]);
      });
      });
      }, "mathjax-editing");

      StackExchange.ready(function() {
      var channelOptions = {
      tags: "".split(" "),
      id: "69"
      };
      initTagRenderer("".split(" "), "".split(" "), channelOptions);

      StackExchange.using("externalEditor", function() {
      // Have to fire editor after snippets, if snippets enabled
      if (StackExchange.settings.snippets.snippetsEnabled) {
      StackExchange.using("snippets", function() {
      createEditor();
      });
      }
      else {
      createEditor();
      }
      });

      function createEditor() {
      StackExchange.prepareEditor({
      heartbeatType: 'answer',
      autoActivateHeartbeat: false,
      convertImagesToLinks: true,
      noModals: true,
      showLowRepImageUploadWarning: true,
      reputationToPostImages: 10,
      bindNavPrevention: true,
      postfix: "",
      imageUploader: {
      brandingHtml: "Powered by u003ca class="icon-imgur-white" href="https://imgur.com/"u003eu003c/au003e",
      contentPolicyHtml: "User contributions licensed under u003ca href="https://creativecommons.org/licenses/by-sa/3.0/"u003ecc by-sa 3.0 with attribution requiredu003c/au003e u003ca href="https://stackoverflow.com/legal/content-policy"u003e(content policy)u003c/au003e",
      allowUrls: true
      },
      noCode: true, onDemand: true,
      discardSelector: ".discard-answer"
      ,immediatelyShowMarkdownHelp:true
      });


      }
      });














      draft saved

      draft discarded


















      StackExchange.ready(
      function () {
      StackExchange.openid.initPostLogin('.new-post-login', 'https%3a%2f%2fmath.stackexchange.com%2fquestions%2f3055540%2fwhen-is-p21-twice-of-a-prime%23new-answer', 'question_page');
      }
      );

      Post as a guest















      Required, but never shown

























      2 Answers
      2






      active

      oldest

      votes








      2 Answers
      2






      active

      oldest

      votes









      active

      oldest

      votes






      active

      oldest

      votes









      4












      $begingroup$

      We have infinitely many primes of the form $$frac{p^2+1}{2}$$ where $p$ is itself prime, if we have infinite many positive integers $k$ such that $$2k+1$$ and $$2k^2+2k+1$$ are simultaneously prime. (in this case, just set $p=2k+1$). The Bunyakovsky conjecture implies that this is the case, so very likely infinitely many examples exist. But I am convinced that the problem is open.






      share|cite|improve this answer











      $endgroup$


















        4












        $begingroup$

        We have infinitely many primes of the form $$frac{p^2+1}{2}$$ where $p$ is itself prime, if we have infinite many positive integers $k$ such that $$2k+1$$ and $$2k^2+2k+1$$ are simultaneously prime. (in this case, just set $p=2k+1$). The Bunyakovsky conjecture implies that this is the case, so very likely infinitely many examples exist. But I am convinced that the problem is open.






        share|cite|improve this answer











        $endgroup$
















          4












          4








          4





          $begingroup$

          We have infinitely many primes of the form $$frac{p^2+1}{2}$$ where $p$ is itself prime, if we have infinite many positive integers $k$ such that $$2k+1$$ and $$2k^2+2k+1$$ are simultaneously prime. (in this case, just set $p=2k+1$). The Bunyakovsky conjecture implies that this is the case, so very likely infinitely many examples exist. But I am convinced that the problem is open.






          share|cite|improve this answer











          $endgroup$



          We have infinitely many primes of the form $$frac{p^2+1}{2}$$ where $p$ is itself prime, if we have infinite many positive integers $k$ such that $$2k+1$$ and $$2k^2+2k+1$$ are simultaneously prime. (in this case, just set $p=2k+1$). The Bunyakovsky conjecture implies that this is the case, so very likely infinitely many examples exist. But I am convinced that the problem is open.







          share|cite|improve this answer














          share|cite|improve this answer



          share|cite|improve this answer








          edited Jan 7 at 10:00









          the_fox

          2,90031538




          2,90031538










          answered Dec 29 '18 at 22:39









          PeterPeter

          48.9k1239136




          48.9k1239136























              1












              $begingroup$

              I propose the following "kind of parametrization". Remark first, as you did, that necessarily $qequiv 1$ mod $4$ (this can be seen directly from the characterization of integers which are sums of squares, see e.g. Samuel's ANT, V-6). Then work in the Gauss ring $mathbf Z[i]$, which is a PID with units $u=pm 1,pm i$. For convenience, write $z'≈z$ for $z'=uz$. It is classically known that $2$ is totally ramified and $q$ splits completely in the Gauss ring, more precisely $2=(1+i)^2≈(1+i)(1-i)$ and $q≈(x+iy)(x-iy)$, where $xpm iy$ are prime elements of $mathbf Z[i]$. Because of unique factorization (up to units), the given equation will be equivalent to $p+i≈(1+i)(x+iy)$, hence our "pseudo-parametrization" will be as follows: 1) Take a prime $qequiv 1$ mod $4$ and decompose it in $mathbf Z[i]$; 2) Solve $p+i≈(1+i)(x+iy)$ in $mathbf Z^2$. Because of symmetry, let us solve only the equation $p+i=-(1+i)(x+iy)$. By identification, $p=y-x$ and $-1=x+y$, so $p=2y+1$. It remains however to check that $2y+1$ is a prime in $mathbf Z$, which is why our method cannot be considered as a genuine parametrization. Although we have just shown that the OP question is essentially equivalent to the Bunyakovsky conjecture evoked by @Peter, I don't know if this gives any new hint.






              share|cite|improve this answer











              $endgroup$


















                1












                $begingroup$

                I propose the following "kind of parametrization". Remark first, as you did, that necessarily $qequiv 1$ mod $4$ (this can be seen directly from the characterization of integers which are sums of squares, see e.g. Samuel's ANT, V-6). Then work in the Gauss ring $mathbf Z[i]$, which is a PID with units $u=pm 1,pm i$. For convenience, write $z'≈z$ for $z'=uz$. It is classically known that $2$ is totally ramified and $q$ splits completely in the Gauss ring, more precisely $2=(1+i)^2≈(1+i)(1-i)$ and $q≈(x+iy)(x-iy)$, where $xpm iy$ are prime elements of $mathbf Z[i]$. Because of unique factorization (up to units), the given equation will be equivalent to $p+i≈(1+i)(x+iy)$, hence our "pseudo-parametrization" will be as follows: 1) Take a prime $qequiv 1$ mod $4$ and decompose it in $mathbf Z[i]$; 2) Solve $p+i≈(1+i)(x+iy)$ in $mathbf Z^2$. Because of symmetry, let us solve only the equation $p+i=-(1+i)(x+iy)$. By identification, $p=y-x$ and $-1=x+y$, so $p=2y+1$. It remains however to check that $2y+1$ is a prime in $mathbf Z$, which is why our method cannot be considered as a genuine parametrization. Although we have just shown that the OP question is essentially equivalent to the Bunyakovsky conjecture evoked by @Peter, I don't know if this gives any new hint.






                share|cite|improve this answer











                $endgroup$
















                  1












                  1








                  1





                  $begingroup$

                  I propose the following "kind of parametrization". Remark first, as you did, that necessarily $qequiv 1$ mod $4$ (this can be seen directly from the characterization of integers which are sums of squares, see e.g. Samuel's ANT, V-6). Then work in the Gauss ring $mathbf Z[i]$, which is a PID with units $u=pm 1,pm i$. For convenience, write $z'≈z$ for $z'=uz$. It is classically known that $2$ is totally ramified and $q$ splits completely in the Gauss ring, more precisely $2=(1+i)^2≈(1+i)(1-i)$ and $q≈(x+iy)(x-iy)$, where $xpm iy$ are prime elements of $mathbf Z[i]$. Because of unique factorization (up to units), the given equation will be equivalent to $p+i≈(1+i)(x+iy)$, hence our "pseudo-parametrization" will be as follows: 1) Take a prime $qequiv 1$ mod $4$ and decompose it in $mathbf Z[i]$; 2) Solve $p+i≈(1+i)(x+iy)$ in $mathbf Z^2$. Because of symmetry, let us solve only the equation $p+i=-(1+i)(x+iy)$. By identification, $p=y-x$ and $-1=x+y$, so $p=2y+1$. It remains however to check that $2y+1$ is a prime in $mathbf Z$, which is why our method cannot be considered as a genuine parametrization. Although we have just shown that the OP question is essentially equivalent to the Bunyakovsky conjecture evoked by @Peter, I don't know if this gives any new hint.






                  share|cite|improve this answer











                  $endgroup$



                  I propose the following "kind of parametrization". Remark first, as you did, that necessarily $qequiv 1$ mod $4$ (this can be seen directly from the characterization of integers which are sums of squares, see e.g. Samuel's ANT, V-6). Then work in the Gauss ring $mathbf Z[i]$, which is a PID with units $u=pm 1,pm i$. For convenience, write $z'≈z$ for $z'=uz$. It is classically known that $2$ is totally ramified and $q$ splits completely in the Gauss ring, more precisely $2=(1+i)^2≈(1+i)(1-i)$ and $q≈(x+iy)(x-iy)$, where $xpm iy$ are prime elements of $mathbf Z[i]$. Because of unique factorization (up to units), the given equation will be equivalent to $p+i≈(1+i)(x+iy)$, hence our "pseudo-parametrization" will be as follows: 1) Take a prime $qequiv 1$ mod $4$ and decompose it in $mathbf Z[i]$; 2) Solve $p+i≈(1+i)(x+iy)$ in $mathbf Z^2$. Because of symmetry, let us solve only the equation $p+i=-(1+i)(x+iy)$. By identification, $p=y-x$ and $-1=x+y$, so $p=2y+1$. It remains however to check that $2y+1$ is a prime in $mathbf Z$, which is why our method cannot be considered as a genuine parametrization. Although we have just shown that the OP question is essentially equivalent to the Bunyakovsky conjecture evoked by @Peter, I don't know if this gives any new hint.







                  share|cite|improve this answer














                  share|cite|improve this answer



                  share|cite|improve this answer








                  edited Jan 7 at 9:14

























                  answered Jan 7 at 8:45









                  nguyen quang donguyen quang do

                  9,0091724




                  9,0091724






























                      draft saved

                      draft discarded




















































                      Thanks for contributing an answer to Mathematics Stack Exchange!


                      • Please be sure to answer the question. Provide details and share your research!

                      But avoid



                      • Asking for help, clarification, or responding to other answers.

                      • Making statements based on opinion; back them up with references or personal experience.


                      Use MathJax to format equations. MathJax reference.


                      To learn more, see our tips on writing great answers.




                      draft saved


                      draft discarded














                      StackExchange.ready(
                      function () {
                      StackExchange.openid.initPostLogin('.new-post-login', 'https%3a%2f%2fmath.stackexchange.com%2fquestions%2f3055540%2fwhen-is-p21-twice-of-a-prime%23new-answer', 'question_page');
                      }
                      );

                      Post as a guest















                      Required, but never shown





















































                      Required, but never shown














                      Required, but never shown












                      Required, but never shown







                      Required, but never shown

































                      Required, but never shown














                      Required, but never shown












                      Required, but never shown







                      Required, but never shown







                      Popular posts from this blog

                      How do I know what Microsoft account the skydrive app is syncing to?

                      When does type information flow backwards in C++?

                      Grease: Live!